Простое доказательство ковариантности уравнений Максвелла

Я читал, что уравнения Максвелла ковариантны относительно преобразований Лоренца, но не могу найти доказательства. Или хотя бы доказательство, понятное человеку, не знающему высшей математики (пожалуйста, не начинайте писать иероглифы в тензорной нотации, потому что я их не понимаю: и обратите внимание, что из-за этой просьбы этот вопрос не дублируется ) . Я попытался сделать простое исчисление, найдя то, что следует. Рассмотрим две стандартные системы С и С (в движении к положительному Икс ) мы используем в теории относительности. Предположим, в С Уравнения Максвелла работают:

{ Е "=" р ε 0 Б "=" 0 × Е "=" Б т × Б "=" мю 0 Дж + ε 0 мю 0 Е т
Тогда я ожидаю, что делая замены
Е "=" ( Е Икс , γ ( Е у в Б г ) , γ ( Е г + в Б у )
Б "=" ( Б Икс , γ ( Б у + в с 2 Е г ) , γ ( Б г в с 2 Е у ) )
р "=" ( 1 ты Икс в с 2 ) γ р
Икс "=" γ ( Икс + в с 2 т )
у "=" у
г "=" г
т "=" γ ( т + в Икс )
ты Икс "=" ты Икс в 1 ты Икс в с 2
ты у "=" ты у γ ( 1 ты Икс в с 2 )
ты г "=" ты г γ ( 1 ты Икс в с 2 )
я должен получить
{ Е "=" р ε 0 Б "=" 0 × Е "=" Б т × Б "=" мю 0 Дж + ε 0 мю 0 Е т
Но если вы это сделаете, то увидите, что это работает только для поперечной составляющей ( у и г ) векторных уравнений (3-е и 4-е, те, что с завихрением). Сосредоточимся, например, на более простом уравнении Максвелла, втором, что в С является: Б "=" 0 . Делая замены, это преобразуется в Б "=" 0 только если Икс компонент × Е + Б т равен нулю. На первый взгляд вы могли бы сказать: «В чем проблема? Это работает из-за 3-го уравнения Максвелла». Проблема в том, что это не загрунтовано! Моя исходная гипотеза, основанная на уравнениях Максвелла со штрихами: уравнения Максвелла без штрихов — это то, что я пытаюсь доказать. Если каким-то образом вы сможете доказать, что продольная компонента штрихованного 3-го уравнения Максвелла правильно преобразуется в незаштрихованное, то доказательство будет работать, но если вы попытаетесь преобразовать × Е "=" Б т ты найдешь × Е "=" Б т только если предположить Б "=" 0 . Другими словами

  • заштрихованное 2-е уравнение стало не заштрихованным 2-м уравнением только в том случае, если продольная компонента не заштрихованного 3-го уравнения работает
  • продольная компонента штрихованного 3-го уравнения стала продольной компонентой не штрихованного 3-го уравнения, только если не штрихованное 2-е уравнение работает

Этот тупик раздражает. Выполняя вычисления, вы можете видеть, что все происходит так же, как и в двух других уравнениях.

  • заштрихованное 1-е уравнение стало незаштрихованным 1-м уравнением только в том случае, если продольная компонента не заштрихованного 4-го уравнения работает
  • продольная компонента штрихованного 4-го уравнения стала продольной компонентой не штрихованного 4-го уравнения, только если 1-е уравнение работает

Я могу написать еще более компактно (здесь числа относятся к уравнениям Максвелла в том порядке, в котором я использовал выше)

  • 2 к 2 если 3 Икс
  • 3 Икс к 3 Икс если 2
  • 1 к 1 если 4 Икс
  • 4 Икс к 4 Икс если 1

Таким образом, вы можете сразу увидеть, что мы находимся на улице без выхода! я мог бы добавить

  • 3 у к 3 у без проблем
  • 3 г к 3 г без проблем
  • 4 у к 4 у без проблем
  • 4 г к 4 г без проблем

Может быть, у меня не было бы проблем с возможной формулировкой уравнений Максвелла? (это не выглядит непомерно трудным, как тензорный подход, но я не пробовал таким образом) Во всяком случае, чтение Резника «Introduzione alla relatività ristretta» заставляет меня думать, что формулировка поля должна быть хороша для этого доказательства, но он явно выполняет вычисления только для у компонент × Е "=" Б т , что является одним из частных случаев, в которых это доказательство работает! Я не могу поверить, что для доказательства инвариантности нам нужно изменить формализм, ведь можно достичь цели с помощью каких-то хитрых уловок, которых я не вижу. Но какой?

Если вы хотите увидеть доказательство этого без тензорных обозначений, то оно есть в статье Эйнштейна по специальной теории относительности 1905 года.
Зачем кому-то хотеть делать это без ковариантной записи?
@ my2cts а) Потому что эта запись проще и поэтому понятна даже тем, кто не знаком с тензорным исчислением. б) Потому что даже физики, а не только математики, могут найти удовольствие (а в некоторых случаях и полезное) в разработке альтернативных доказательств.

Ответы (2)

Кажется, вы фактически доказали инвариантность: учитывая, что полный набор уравнений Максвелла верен в одной системе отсчета, они верны и в другой системе отсчета.

Но вам это не нравится, потому что вы хотите каким-то образом сделать это независимо для одного из уравнений за раз. Это невозможно, потому что только набор уравнений Максвелла вместе обладает свойством инвариантности. Вот в чем действительно глубокая суть всего этого: закон Кулона, или закон Фарадея, или любая другая часть сама по себе зависит от системы отсчета. Только при унификации Максвелла (включая член смещения) вы получаете единый инвариантный набор. И эта инвариантность требует специальной теории относительности, а не ньютоновской теории относительности.

Первый абзац, кажется, смешивает формоинвариантность при калибровочных преобразованиях с формоинвариантностью при преобразованиях Пуанкэра.
Я не понимаю начало вашего ответа: я не доказал инвариантность (это моя проблема!). Я тоже не понимаю конец вашего ответа: я использую не преобразования Галилея, а преобразования Лоренца. Еще меньше я понимаю критику «вы хотите как-то сделать это независимо для одного из уравнений за раз». Это совершенно неправильно, я рассматривал уравнения Максвелла все вместе, поэтому свел их в систему и в попытках доказать инвариантность одних пытался использовать другие (то, что трансформируются только поперечные компоненты, случайно).
Так в чем проблема? Вы можете показать, что все они инвариантны, если они все инвариантны. Это прекрасное доказательство.
@BobJacobsen Моя проблема в том, что действия путем замены и упрощения (даже с использованием других уравнений) не работают. Четыре из 8 уравнений (1,2,3x,3y,3z,4x,4y,4z) преобразуются в одиночку, и я надеялся, что это отправная точка для преобразования других. Но для преобразования других в незаштрихованные нам не нужны 3y, 3z, 4y или 4z (это было бы неплохо!) нам нужны другие незаштрихованные уравнения 1, 2, 3x или 4x, но я не могу использовать их, потому что моя исходная гипотеза была загрунтована.

Я нашел способ решить проблему: давайте начнем с явного написания 8 уравнений

{ ( а ) Е Икс Икс + Е у у + Е г г "=" р ϵ 0 ( б ) Б Икс Икс + Б у у + Б г г "=" 0 ( с ) Е г у Е у г "=" Б Икс т ( г ) Е Икс г Е г Икс "=" Б у т ( е ) Е у Икс Е Икс у "=" Б г т ( ф ) Б г у Б у г "=" мю 0 Дж Икс + ϵ 0 мю 0 Е Икс т ( г ) Б Икс г Б г Икс "=" мю 0 Дж у + ϵ 0 мю 0 Е у т ( час ) Б у Икс Б Икс у "=" мю 0 Дж г + ϵ 0 мю 0 Е г т
Выполняя замены ответа, мы можем переставить таким образом (я пишу [ А ] Икс для А Икс )
{ ( а ) Е р ϵ 0 "=" в [ × Б мю 0 Дж ϵ 0 мю 0 Е т ] Икс ( б ) Б "=" в с 2 [ × Е + Б т ] Икс ( с ) [ × Е + Б т ] Икс "=" в Б ( г ) Е Икс г Е г Икс "=" Б у т ( е ) Е у Икс Е Икс у "=" Б г т ( ф ) [ × Б мю 0 Дж ϵ 0 мю 0 Е т ] Икс "=" в с 2 ( Е р ϵ 0 ) ( г ) Б Икс г Б г Икс "=" мю 0 Дж у + ϵ 0 мю 0 Е у т ( час ) Б у Икс Б Икс у "=" мю 0 Дж г + ϵ 0 мю 0 Е г т
Мы видим, что (г), (д), (ж) и (з) автоматически переходят в нештрихованное уравнение (поэтому в ответе я написал, что 3 у , 3 г , 4 у и 4 г превращаться в 3 у , 3 г , 4 у и 4 г без проблем), а остальные четыре уравнения я упорядочил удобным образом: задача состояла в том, чтобы доказать инвариантность этих. Но после написания системы вышеописанным способом доказательство простое:

  • умножение (с) на в с 2 и суммируя с (b), получаем Б "=" 0 (т.е. не заштриховано (б) первой системы), что с (б) дает Е г у Е у г "=" Б Икс т (т.е. не грунтованная (с) первой системы)

  • умножение (f) на в и суммируя с (а), мы получаем Е "=" р ε 0 (т.е. не заштриховано (а) первой системы), что с (а) дает Б г у Б у г "=" мю 0 Дж Икс + ϵ 0 мю 0 Е Икс т (т.е. не грунтованная (f) первой системы)

Доказательство окончено. Если бы я нашел это доказательство в книге по электродинамике, я бы сэкономил много времени. Жаль, что этого интересного доказательства нет в литературе (не все умеют обращаться с тензорами). В книге Резника показано, как преобразовать (d), но это только самый простой случай.